lunes, 4 de abril de 2016

¿Cuánto mide un metro?

Leyendo el libro Quantum computing since Democritus, de Scott Aaronson, me encuentro con un divertido hecho matemático que no conocía, y que comparto con vosotros (supongo que se tratará de un teorema con alguna denominación común, pero el libro no la da, y no la he encontrado).
.
Se trata del hecho de que, si descomponemos en una serie infinita de intervalos un segmento de la recta de los números reales -p.ej., el segmento (0,1)-, ese conjunto de intervalos podemos reordenarlos de tal forma cubran TODA la recta de los números reales, en su infinita longitud.
.
Aquí va una versión de andar por casa de la prueba (basada en lo poco que dice el libro al respecto; disculpas a los entendidos si meto la pata en algún detalle, y agradecido de antemano por las posibles correcciones):
.
En primer lugar, descomponemos el segmento (0,1) de la manera siguiente: primero tomamos el intervalo abierto que va de 0 a 1/2 (o sea, 1/21); después tomamos el intervalo que va de 1/2 a 3/4 (o sea, un intervalo justo a continuación, y con longitud 1/4 = 1/22); después, el intervalo entre 3/4 y 7/8 (idem, con longitud 1/8 = 1/23); luego el intervalo entre 7/8 y 15/16 (un intervalo de longitud 1/16 = 1/24), y así sucesivamente. O sea, empezamos tomando la mitad del intervalo (0,1), luego la mitad de lo que queda, luego la mitad de lo que queda, y así hasta el infinito.
.
En segundo lugar, recordemos que el conjunto de los números racionales tiene dos propiedades muy importantes: es numerable (podemos formular una función que asigne para cada número natural un número racional, de tal forma que no sobre ningún número racional) y es denso (entre dos números reales cualesquiera hay infinitos números racionales, tan próximos al primero como queramos).
.
A partir de la primera propiedad podemos hacer lo siguiente: formulemos una numeración cualquiera de los números racionales, de tal forma que los tengamos ordenados del primero al último: q1, q2, ..., qn,... (nota: no se trata de ordenarlos "de mayor a menor", lo que obviamente no puede hacerse, pues no hay un número racional que sea el menor de todos, ni el mayor, ni el siguiente a un número racional dado; sino numerarlos según cualquier función definible entre los números naturales y los racionales, tal como la famosa enumeración de Cantor -ver imagen-).
.
En tercer lugar, lo que hacemos es sencillamente asignar al número racional que va en el i-ésimo lugar de esa numeración, el segmento i-ésimo de la serie de segmentos que habíamos sacado del intervalo (0,1), y colocamos ese segmento sobre la recta de los números reales, de tal modo que su centro coincida con el número racional i-ésimo.
.
Esto implica que alrededor de cada número racional habremos colocado un intervalo de extensión mayor que cero. Sólo queda por comprobar que no nos hemos dejado ningún "hueco" en la recta real, de tal forma que la suma de esos huecos sea tan grande que los intervalos que hemos colocado no la cubran entera, sino sólo una parte finita (por ejemplo, su longitud "original" igual a 1).
.
Para esto aplicamos, en cuarto lugar, la propiedad de la densidad de los números racionales. Vamos a descomponer la prueba en dos casos. Primero, no podemos habernos dejado ningún intervalo de la recta real sin cubrir, es decir, un conjunto (a,b), donde a y b son números irracionales, y donde no haya colocado ningún fragmento de los que sacamos del segmento (0,1). Esto es obvio, porque entre los números a y b habrá infinitos números racionales, y cada uno de ellos tendrá colocado a su alrededor un intervalo de los que hemos tomado.
.
Segundo, no podemos habernos dejado ningún número irracional "suelto": si hubiera sido así, tendríamos que el número irracional que nos hemos dejado (p.ej., el raíz de 2 de la imagen) tendría un intervalo a la derecha y otro a la izquierda, de los cuales sería el límite. Pero por construcción de nuestra serie de segmentos, tenemos que la longitud de todos los intervalos sacados del segmento (0,1) es igual a un número racional, y al haberlos colocado de tal forma que justo en su centro haya un número racional, la distancia entre ese número y el borde del intervalo es justo la mitad del intervalo, una medida que también es un número racional, de modo que el límite de los intervalos no puede coincidir nunca con un número irracional.
.
Así pues, todos los números irracionales de la recta real deben estar dentro de algún intervalo de los que hemos recolocado de esta manera tan ingeniosa.
.
¿Cuánto mide, por tanto, "realmente" el segmento (0,1)? Pues depende: no tiene una medida "real" y "absoluta", sino que depende de en qué orden enumeremos los segmentos. Lo que demuestra la construcción que acabo de ofrecer es que hay algún modo de ordenar los infinitos segmentos en que descompusimos el segmento (0,1) de tal modo que la longitud que nos sale es infinita. QED.
.
.
Por cierto, la misma prueba sirve también para demostrar que el intervalo (0,1) -o, para el caso, cualquier otro intervalo del tamaño que deseemos, p.ej., el intervalo (5,000000001, 5,000000002)- podemos cortarlo en trozos que, recolocándolos, cubran cualquier otra longitud que deseemos -p.ej., enumerando el conjunto de números racionales del intervalo (2000, 6000), de modo que hayamos cubierto una longitud de 4000 unidades-.
.
Es decir, que una determinada extensión (un metro, p.ej.), nos puede dar de sí todo lo que queramos. Lo que venían a hacer nuestras madres con el sueldo familiar, o mi tocayo más famoso con los panes y los peces, vaya.

49 comentarios:

  1. Sutil manera esta de colocar los infinitos y numerables intervalos de medida 1/2 , 1/4 , 1/8…”sobre” el conjunto numerable de racionales de manera que no haya un solo numero real que pueda decir que no pertenezca a uno de esos segmentos (cubriendo pues toda la recta real). ¡¡Con un segmento que mide 1 somos capaces de cubrir toda la recta que mide infinito !!. Es una de esas verdades matemáticas que intuitivamente sorprenden, cuando trabajamos con infinitos.

    Pero respeto a la “multiplicación de los panes y los peces” la paradoja matemática que más se ajusta es la de Banach-Tarski ( https://es.wikipedia.org/wiki/Paradoja_de_Banach-Tarski ), también tiene que ver con la teoría de la medida y cuando se trabaja con correspondencias entre conjuntos infinitos. Recuerdo que por la facultad donde estudiaba (hace tanto !) escuché alguna que otra vez este chistoso comentario propio de matemáticos: “Si Jesús multiplicó los panes y los peces es porque usaba y conocía el axioma de elección”, creo que provenía de una cita de algún matemático famoso que no recuerdo.( Tal axioma es necesario para dar cuenta de esos aspectos propios de los “conjuntos paradoxales” https://en.wikipedia.org/wiki/Paradoxical_set )

    Me gustó que compartiese algo de matemáticas tan aparentemente contraintuitivas.

    Un saludo

    ResponderEliminar
  2. Gracias, Enric. Pensé relacionar el artículo con la paradoja de Banach-Tarski, pero lo dejé por no complicar más (además, esa demostración ya sí que es inasequible para mi -in-capacidad). Un saludo

    ResponderEliminar
  3. Don Jesús,

    ¿Dónde está el truco? En la paradoja Banach-Tarski no es posible asignar una medida definida a los trozos, pero este no parece ser el caso aquí. El primer trozo mide claramente medio metro, el segundo un cuarto, etc. He leído el capítulo Aaronson, pero no he conseguido más claridad (de hecho, ni siquiera habría entendido por mí mismo el teorema).

    ResponderEliminar
  4. Hola, Perplejo. No hay truco. Lo que falla es nuestra intuición de que una serie infinita de sumandos tiene una suma bien definida. Depende de cómo estén ordenados los sumandos

    ResponderEliminar
  5. Perplejo,

    La suma de infinitos términos 1/2 + 1/4 + 1/8 +… es convergente y siempre es 1. Tal suma no depende del orden con que sumemos, y no tiene las típicas paradojas de las series divergentes que se suelen establecer como divertimento matemático. La suma de esa serie siempre es 1, se ordene como se ordene la serie.
    Pero el cardinal del conjunto de números reales que pertenecen al intervalo (0,1), o a la unión de los números reales que pertenecen a esos “segmentos” de longitudes 1/2, 1/4 , 1/8 , … ¡NO ES 1!, sino que es el mismo que el de toda la recta real. Recuerde que el cardinal del intervalo (0,1) es el mismo que el cardinal de todo R, como ya mostró Cantor.
    Establecer una correspondencia entre cualquier numero de la recta real y los números reales pertenecientes a los infinitos intervalos que se han considerado, (observa que el el numero de intervalos es infinito numerable, pero el de numeros reales que hay en cada uno de esos intervalos es no numerable), no es lo mismo que establecer que la suma de las LONGITUDES de esos infinitos intervalos sea la longitud infinita de toda la recta real.

    Un saludo.

    ResponderEliminar
  6. Enric,

    Muchas gracias por responder. En efecto, no es lo mismo establecer la correspondencia de puntos que la igualdad de longitud. Sin embargo, eso parece ser precisamente lo que se logra aquí: con el sistema indicado, los trozos de un segmento de un metro podrían cubrir completamente un segmento de diez kilómetros.

    Saludos

    ResponderEliminar
  7. Jesús,

    Es cierto que hay sumas infinitas cuyo resultado no está bien definido. Sin embargo, y como ya ha indicado Enric, este no parece el caso de 1/2+1/4+1/8..., que converge necesariamente en 1. Lo más parecido a una respuesta que consigo para calmar mi perplejidad es pensar que "en la naturaleza" no hay rectas, ni puntos, ni nada, y que por ahí se explica el enigma, pero la verdad que no me satisface.

    Un saludo

    ResponderEliminar
  8. Enric, tú sabes más que yo de esas cosas. Admito mi perplejidad

    ResponderEliminar
  9. Perplejo,

    Pero no hacemos un recubrimiento de toda la recta Real en sentido topológico (https://es.wikipedia.org/wiki/Recubrimiento_(matem%C3%A1tica)).

    Los intervalos [0,1/2), [1/2,1/4), [1/4,1/8)… no contienen ni el 3, ni pi…y por tanto NO recubren R. Que coloquemos a cada uno de estos intervalos sobre los números racionales (en virtud que tenemos infinitos pero numerables al igual que de números racionales) no significa que recubramos la recta Real, solo significa que hemos hecho una biyección entre cada número racional y el conjunto infinito numerables de esos intervalos. Y en virtud de eso, somos capaces de ESTABLECER también una biyección de cada número Real con un número real de cada uno de los intervalos, pero eso no significa, repito, que hayamos recubierto (en sentido topológico) la recta. Observa que todo número real tiene asociado un solo número de un “intervalo de esos”, pero no todo “intervalo de esos” tiene asociado un solo número real, de manera que no hay una correspondencia biyectiva entre cada intervalo y cada número real, es biyectiva sólo entre cada número real y los números pertencientes al conjunto de esos intervalos ( es decir los números del intervalo (0,1). )

    Al establecer la biyección creemos que recubrimos la recta, o podemos recubrirla, pero eso es falso. Por ejemplo: que el intervalo (0,1) tenga el mismo cardinal que toda la recta real (podemos establecer una biyección entre todos los números que hay en el intervalo (0,1) y todos los reales) no significa que (0,1) recubra la recta real. En cambio el conjunto infinito de intervalos (z-1, z] con z entero Sí que recubria todo R.

    Un saludo.

    ResponderEliminar
  10. Enric
    La cuestión no es sólo la biyección entre los infinitos intervalos del segmento (0,1) y el conjunto de números reales, sino la TRASLACIÓN de cada uno de aquellos intervalos a un lugar determinado de la recta real. Es decir, si acaso lo que hacemos es una biyección entre el primer conjunto de intervalos, y OTRO conjunto de intervalos (los que hemos identificado ahora alrededor de cada número racional). La cuestión es, ¿hay alguna parte de la recta (sea una parte que consista en un intervalo, o en un número "suelto") que no esté contenida en ninguno de los intervalos de este segundo conjunto? Y el teorema parece mostrar que no: es decir, con los segmentos sacados de (0,1) habríamos cubierto toda la recta. Puede que haya algún fallo, pero yo no lo veo.

    ResponderEliminar
    Respuestas
    1. Sean E1, E2, …En los infinitos intervalos que cubren (0,1) pero NO Cubren R.

      Sean Q1, Q2,…Qn los infinitos intervalos que están en biyección con los Ei y son traslaciones de los Ei sobre los racionales y que SÍ cubren R como muestra el teorema. Eso es una verdad matemática, más que una paradoja, aunque sea contraintuitivo.

      ¿Eso sorprende? . Pues lo que intento decir es que no debería sorprender más que decir que hay la misma cantidad (cardinal) de números en (0,1) que en toda la recta. Precisamente esa es la definición de conjunto infinito, la de que “el todo” se puede poner en biyeccion con una parte de ese todo. Si esto es así, no es tan extraño que podamos cubrir todo R con trozos del segmento (0,1).

      ¿Significa eso que si podemos cubrir con trozos que juntos miden 1 toda la recta entonces toda la recta mide 1?. Pues no.
      La suma de las longitudes de los Ei es 1 y está bien definida pues son disjuntos dos a dos. Pero la suma de las longitudes de los Qi no está bien definida, ya que para cualquier Qi que cojamos hay infintos Qj que lo interseccionan, pues en cada racional x que pertenece a Qi hay un intervalo Qx diferente a Qi del cual x es centro. Es decir, no son disjuntos dos a dos los Qi i la medida no está bien definida, la medida m no se verifica que m(A U B) es igual m(a) + m(b), si A y B no son disjuntos.
      Es obvio que la longitud de Ei es igual a lo longitud de Qi (la medida se conserva por traslación).Intuitivamente pensamos que la suma de longitudes de los Ei ha de ser que la de las Qi, pero por construcción, y precisamente para poder “llenar” toda la recta, los Qi no son disjuntos, ya que dentro que cada Qi hay infinitos Qj que lo interseccionan. Y si hacemos transformaciones que los pongan de manera disjunta, por ejemplo colocando cada intervalo Qi sobre los números naturales, entonces su medida “vuelve” a ser 1, y dejan de cubrir toda la recta.

      Un saludo.

      Eliminar
  11. No encuentras un teorema que enuncie el divertido hecho porque tal teorema no existe. La demostración no es tal, a pesar de las apariencias. Una vez que estableces la biyección entre los intervalos que has sacado del (0,1) y los números racionales tienes que mostrar cómo cubres todos los irracionales. Para ello necesitas mostrar qué intervalo lo hace. La manera de hacerlo será aproximarse al irracional a través de un límite de racionales y aquí te van a faltar intervalos de la longitud adecuada: a medida que te acerques al irracional habrás ido gastando muchos intervalos y los que te queden van a ser más pequeños que la distancia que te queda por recorrer hasta llegar al irracional.

    Detalles:

    1. Lo anterior no es estrictamente cierto para todos los irracionales que quieras aproximar, podrás hacerlo así con unos pocos (acaso un conjunto contable de ellos), pero no con los demás (recuerda que son incontables). Es decir, que no puedes mostrar cómo cubrir un conjunto incontable de racionales cuya medida, por lo demás, será infinita.

    2. La densidad de los racionales implica que entre dos irracionales cualesquiera siempre hay algún racional (y la de los irracionales, que entre dos racionales hay algún irracional). Esto nos suena paradójico y pareciera con ello que hay los mismos de ambos, pero es lo que tiene el infinito. Lo que pasa es que cuando cojo un par de números, siempre son finitos y la distancia entre ellos también es finita. Si quieres cubrir un irracional con un parche centrado en un racional tienes siempre una distancia finita y tienes que mostrar que ese parche cubre esa distancia. El argumento de que los racionales son densos no sirve para mostrar que siempre tienes ese parche de entre los sacados del intervalo (0,1), puesto que puedes haber usado ya los que son tan grandes como esa distancia.

    ResponderEliminar
    Respuestas
    1. ·”Es decir, que no puedes mostrar cómo cubrir un conjunto incontable de racionales cuya medida, por lo demás, será infinita.”

      No entiendo. Los racionales es un conjunto contable. Y la medida de todos los racionales es nula. Supongo que te refieres a los irracionales que efectivamente su conjunto no es de medida nula (no es nulo). Su medida de Lebesgue es infinito.

      un saludo.

      Eliminar
    2. Efectivamente, ahí debía poner irracionales. Perdón por el error y gracias por la corrección.

      Eliminar
  12. Gracias, José Luis, pero no entiendo por qué no es válido el argumento que doy sobre por qué ningún número real "suelto" puede caer fuera de alguno de los intervalos trasladados.

    ResponderEliminar
  13. Hola a todos.

    Tengo poco que añadir a las dudas que está planteando Jesús, pero intentaré resumir mi postura.
    Tal como yo lo veo,probar el "teorema" (lo entrecomillo para indicar que su validez está sujeta a duda) equivale a probar que no hay ningún hueco entre los intervalos, una vez recolocados, y esto puede probarse muy sencillamente:
    -Los extremos de cada intervalo, una vez recolocados en la recta, coinciden con un número racional.
    -Sobre cada número racional hay un intervalo.
    -Por tanto, ningún extremo puede quedar libre. Es decir: a ningún extremo le sigue un hueco, puesto que a todos le sigue la mitad de otro intervalo.

    Por decirlo gráficamente, todos los intervalos se solapan necesariamente, porque el extremo de cada uno es el centro de otro.

    Un saludo.

    ResponderEliminar
    Respuestas
    1. El asunto de la biyección entre los puntos del intervalo y los puntos de la recta, por cierto, me parece que no tiene demasiado que ver con el "teorema" que nos ocupa. La única biyección necesaria, hasta donde yo creo entender, es la que se establece entre los racionales de la recta real (o los de un segmento de diez kilómetros, para el caso) y el punto central de cada uno de los trozos de ese segmento inicial de un metro.

      Eliminar
  14. A ver, señores, que no es el caso de tener opiniones, que esto es matemáticas. No se trata de ver que "una vez colocados, no queda fuera ningún irracional", sino de mostrar cómo los se colocan para que esto no pase. El argumento de la entrada no muestra cómo se hace, se saca de la manga que por establecer la biyección entre intervalos y números racionales ya están cubiertos de la manera que interesa y no es así, tiene que ser una manera muy precisa en la que no pase lo que señalo en el comentario anterior, y es imposible que no pase por las razones que he dicho.

    ResponderEliminar
  15. José Luis, por supuesto que no se trata de opiniones, sino de demostraciones; lo que pasa es que somos un poco cenutrios y no entendemos la demostración, y como tenemos la opinión de que otros argumentos parecen apuntar en sentido contrario, deseamos salir de esta situación de perplejidad con argumentos que sí entendamos. En mi caso, el argumento que doy en la entrada, según el cual no puede quedar ningún intervalo sin cubrir en la recta real, ni tampoco ningún número irracional que no esté dentro de alguno de esos intervalos, ese argumento, digo, tira de mi sentimiento de convicción de manera irresistible, qué le voy a hacer; me gustaría ver claramente por qué ese argumento está mal, pero a estas horas todavía no lo he visto.

    ResponderEliminar
  16. Yo ha he visto el error de mi "demostración" y quizá de la de Jesús. He tenido la epifanía mientras cenaba el gazpacho.
    Yo he dado por supuesto que el hecho de que ningún intervalo vaya seguido por un hueco implica que toda la recta ha sido cubierta. Esto no es cierto. Es compatible que ningún intervalo tenga al lado un hueco y, sin embargo, el conjunto de todos los intervalos sí puede estar seguido por un hueco. De hecho, esto es justo lo que ocurre en el segmento inicial. Al lado de cada trozo hay otro trozo (al lado del de 1/2, el de 1/4, etc.) y, sin embargo, esto no impide que el segmento completo no sea seguido por otro trozo, sino por un hueco.

    No hay, por tanto, tal recubrimiento.

    Un saludo

    ResponderEliminar
  17. Jose Luis Ferreira,

    · “No se trata de ver que "una vez colocados, no queda fuera ningún irracional", sino de mostrar cómo los se colocan para que esto no pase.”

    Parece que usted es de la escuela constructivista:

    https://es.wikipedia.org/wiki/Constructivismo_(matem%C3%A1ticas)

    · “tiene que ser una manera muy precisa en la que no pase lo que señalo en el comentario anterior, y es imposible que no pase por las razones que he dicho.”

    Lo que no entiendo es desde las razones que dice demasiado intuitivamente definidas, cómo es que acepta que el conjunto de los números racionales es numerable. Si entre dos racionales hay infinitos racionales, como numerándolos vamos a conseguir numerarlos todos si no vamos a poder “avanzar”. Cuando consigo numerar unos cuantos billones de racionales mayores sigo estando igual "de cerca del primer racional numerado".

    Del mismo modo, hay series cuyos términos tienden a cero (cada vez se hacen más pequeños) pero que divergen (por ejemplo la serie armónica:

    https://es.wikipedia.org/wiki/Serie_arm%C3%B3nica_(matem%C3%A1tica)

    es decir, que aunque vamos sumando trozos cada vez más pequeños de esas series podemos sobrepasar cualquier número por muy grande que este sea.

    un saludo

    ResponderEliminar
  18. Algunas puntualizaciones que repito :

    Sea E1,E2,E3….EN los infinitos intervalos numerables de (0,1) que estamos considerando.

    Sea Q1,Q2,Q3…QN los infinitos intervalos numerables que se encuentran centrados sobre los racionales al modo también considerado. Hay una biyección entre los Ei y los Qi por traslación.

    1) Los Ei son disjuntos y su unión tiene una medida bien definida (https://es.wikipedia.org/wiki/Teor%C3%ADa_de_la_medida ) , en concreto es 1.

    2) Los Qi no son disjuntos y su unión no tiene bien definida la medida. Dentro de cada Qi hay infinitos racionales con su intervalo Qx correspondiente, es decir dentro de cada Qi hay infinitos Qj que lo interseccionan.

    3) Si en vez de colocar los intervalos Ei sobre los racionales los colocásemos sobre los naturales tendríamos una unión de conjuntos disjunta, y la medida estaría bien difinida, seria nuevamente 1. Pero no recubriria R

    4) Los Ei no cubren la recta, solo cubren el intervalo abierto (0,1)

    5) Los Qi si cubren la recta, pero la unión de todos los Qi que cubren todo R y estan en correspondencia con el intervalo (0,1), no tiene una medida obtenible desde la medidad de cada Qi por separado.
    Para los que estén interesados en ver como es posible que podamos construir desde un conjunto infinito de intervalos de (0,1) medibles, un conjunto no medible, puede consultar el conjunto de Vitali (que es un resultado no constructivo, por si interesa a Jose Luis Ferreira):

    https://es.wikipedia.org/wiki/Conjunto_de_Vitali


    un saludo.

    ResponderEliminar
  19. Tanto la existencia de los conjuntos no medibles como la paradoja de la entrada son las primeras cosas que aprende uno cuando estudia teoría de la medida. El argumento que resuelve la paradoja (encontrando su falsedad en el argumento) no depende de que sea uno constructivista o no.

    En mi anterior mensaje intentaba dar la intuición de por qué el teorema no es cierto. A continuación especifico mejor la trampa del argumento de la entrada. Es más técnica y creo que menos intuitiva, pero igual os gusta más.

    La parte del argumento que dice que cualquier irracional tiene que estar como mucho en el borde de un intervalo centrado en un racional es la parte que falla. No podemos decir eso. Lo más que podemos decir es que nos acercamos a un irracional mediante intervalos centrados en racionales que se van aproximando al irracional. No hay garantía de que el límite de esa secuencia de intervalos tenga que tener un extremo racional, que es lo que generaría la contradicción. El límite de una secuencia de números racionales (el extremo de cada intervalo en la secuencia) puede muy bien ser un número irracional.

    ¿Ahora sí?

    ResponderEliminar
  20. Gracias, José Luis, creo que ahora sí lo he visto

    ResponderEliminar
  21. Jose Luis Ferreira,

    · “Tanto la existencia de los conjuntos no medibles como la paradoja de la entrada son las primeras cosas que aprende uno cuando estudia teoría de la medida.

    Pues yo la paradoja de la entrada es la primera vez que me la encuentro y eso que estudié “teoría de la medida” hasta casi odiarla. ¿Conoce alguna referencia, apuntes o libro sobre la teoría de la medida que hable de esta “paradoja de la entrada”?

    · “El argumento que resuelve la paradoja (encontrando su falsedad en el argumento) no depende de que sea uno constructivista o no.

    Claro que no. Pero yo “le acusaba” de constructivista en tanto exigía que “construyésemos” no sé muy bien qué, que Estabamos haciendo matemáticas y no dando mera opinión.
    Por otro lado, yo no veo paradoja alguna a resolver, sólo veo una verdad matemática contraintuitiva, típica del uso de conjuntos infinitos y tal.

    ·Y explico porque no veo paradoja alguna:

    Sea E1,E2,E3….EN los infinitos intervalos numerables de (0,1) que estamos considerando.
    Sea Q1,Q2,Q3…QN los infinitos intervalos numerables que se encuentran centrados sobre los racionales al modo también considerado. Hay una biyección entre los Ei y los Qi por traslación.

    Lo que se está diciendo es que hemos distribuido los infinitos intervalos Ei, formando los intervalo Qi (a los números del primer intervalo E1 les hemos sumado “el primer” racional numerado q1, a los del segundo intervalo E2 el segundo racional q2…), DE TAL MANERA QUE NO HAY NINGÚN INTERVALO DE LA RECTA (POR MUY PEQUEÑO QUE SEA) QUE PUEDA DECIR QUE EN SU SENO NO HAY ELEMENTOS DE LOS Qi.
    Cosa que no sería el caso, por ejemplo, si hubiésemos distribuido todos los Ei centrándolos en los naturales, en que fácilmente podríamos dar un intervalo [a,b] que no tiene puntos de los Qi.
    Es en ese sentido en el que estamos cubriendo la recta real (no en “el topologico” como dije anteriormente), en el sentido de que la distribución de los Ei sobre los racionales hace imposible dar un intervalo “ que esté libre” de Qi. Y es en ese sentido que es contraintuitivo, pues lo que cabría esperar es que por mucho que distribuyésemos los Ei (que apenas miden todos juntos 1) por toda la recta, seguro que habría trozos de la recta no “tocados” por los “trozos” Ei.

    Si usted cree que hay algún intervalo, por muy pequeño que sea, que no quede “tocado” por los Qi , me gustaría conocerlo. “Construya” uno ;-) .

    Un saludo.

    ResponderEliminar
    Respuestas
    1. No sé por qué habría de construir un intervalo así. La falsedad de la supuesta demostración no depende de eso, sino de que no puedes mostrar que cubres todos los irracionales, tal y como señalo en el mensaje anterior (o, por lo menos, todos menos acaso un número contable de ellos). Recuerda que la medida de los racionales es cero y que, por tanto, sin cubrir los irracionales te queda todo por cubrir. De hecho, cubres unos pocos, exactamente un conjunto de medida 1 (la del intervalo 0,1), así que te queda toda la recta menos un conjunto de intervalos que entre todos suman 1.

      Eliminar
    2. Claro que no.

      Pero el teorema, tal como lo interpreto yo, lo que viene a decir es que contra toda intuicion resulta que somos capaces de distribuir infinitos pedazos que miden uno, sin que quede un infimo intervalo de toda la recta que pueda decir que no es" tocado" por alguno de esos pedazoss tan sutilmente trasladados.
      Lo que se hace en el fondo no es recubrir toda la recta, sino mostrar que colocando los numeros reales que hay en el intervalo (0, 1) de manera estrategica relacionamos cualquier numero de la recta con los del intervalo (0, 1), y como decia comentarios atras esta verdad matematica contraintuitiva lo es en el mismo sentido que nos puede resultar contraintuitivo que haya igual cardinal en (0, 1) como en todo R.

      Eliminar
  22. Enric
    Creo que ya me ha quedado claro, con las últimas explicaciones de José Luis, que en realidad no podemos demostrar que todo número real de la recta esté comprendido dentro de uno de esos intervalos, así que seguramente lo que tenemos es que hay un conjunto no denumerable de números irracionales que no lo estarán. Eso sí, la forma en la que se "escapan" es más sutil de lo que yo podía imaginar: no es que sean el límite de un intervalo abierto, sino que son el límite de una sucesión de intervalos abiertos, de ninguno de los cuales son el límite.
    Por cierto, otro dato para que no disminuya la perplejidad: en la traslación que se sugiere al principio de la entrada, el conjunto de irracionales que no quedan comprendidos en ningún intervalo tendría una longitud infinita (aunque están todos ellos separados unos de otros); en cambio, si hacemos una traslación de los intervalos del segmento (0,1) a cualquier otros segmento finito (p.ej., el (0,10)), lo que tendremos será que el conjunto de los irracionales no cubiertos por ningún intervalo en esa traslación mide justo 9, aunque son un conjunto de la misma cardinalidad que el conjunto de irracionales no cubiertos en el primer caso, y aunque ellos tampoco están en contacto unos con otros.
    Saludos

    ResponderEliminar
  23. Dándole vueltas a la paradoja, encuentro que todavía hay hueco (valga la broma) para la perplejidad. La cuestión, muy brevemente, sería esta:

    Una vez trasladados los infinitos intervalos, los irracionales de la recta real parecería que sólo pueden haber quedado en dos situaciones: o bien están incluidos en un intervalo, o bien son el límite inalcanzable de alguna sucesión de intervalos (o de una sucesión de sucesiones, o de una sucesión de sucesiones de sucesiones, o de una...).

    Si lo anterior fuera cierto, sin embargo, habría que concluir que los irracionales inalcanzables son tantos como las sucesiones de intervalos de las que son límite; es decir, serían contables, como los racionales, y por lo tanto su medida sería 0.

    Es muy probable que lo anterior sea sólo resultado de mi perfecta incompetencia matemática, pero no quería dejar de anotarlo por si pudiera dar origen a algún comentario interesante.

    Un saludo

    ResponderEliminar
  24. ¡Con lo tranquilo que me había quedado yo!!!!!

    ResponderEliminar
    Respuestas
    1. Jajaja, la culpa es tuya, por traer el tema! Esto es un sinvivir!

      Eliminar
    2. Quedaos tranquilos:

      "o bien son el límite inalcanzable de alguna sucesión de intervalos."

      No es eso lo que se ha dicho, sino que el límite de extremos racionales de una sucesión de intervalos no tiene por qué tener un extremo racional. Y eso se ha dicho para señalar un fallo en el argumento de la entrada. No se ha dicho que puedas definir una sucesión para acercarte a cada número irracional. De hecho no puedes, que es lo que decía en mi primer comentario, ya que te vas a quedar enseguida sin intervalos.

      Es decir, para cualquier intento de cubrir la recta con los intervalos trasladados desde el [0,1] te encontrarás con que un irracional o bien está cubierto por un intervalo, o bien es el límite de los extremos de una sucesión de intervalos o bien ni siquiera puedes aproximarlo con sucesiones de intervalos.

      Eliminar
  25. Este comentario ha sido eliminado por el autor.

    ResponderEliminar
  26. Perplejo,

    · “los irracionales inalcanzables son tantos como las sucesiones de intervalos de las que son límite; es decir, serían contables, como los racionales, y por lo tanto su medida sería 0”

    Piense en la notación decimal de números reales. Los irracionales tienen infinitas cifras que no forman período, los racionales sí. Es fácil ver que todo expresión decimal de un número no es sino una sucesión de Cauchy 1/3 = 1/1 + 3/10 + 3/100… y PI = 3/1 +1/10 +4/100 +…. La primera es definible “algorítmicamente”, la segunda no. Hay pues una “cantidad” no numerable de sucesiones de Cauchy. Es cierto que el conjunto de todas las sucesiones de Cauchy que puedan establecerse desde una algoritmo que permita calcular sus términos o desde cualquier otro modo de definirla es numerable, es decir, sólo una cantidad numerable de sucesiones de Cauchy son “definibles algorítmicamente”. Dicho de otro modo, hay sucesiones de Cauchy que son tan “indefinibles” como las muchas cifras decimales de los números irracionales. Que por ejemplo el número “e” se pueda definir como el límite de una sucesión “definible algorítmicamente” (por ejemplo expresando el término general de la sucesión) no significa que todos lo irracionales, ni muchísimo menos, puedan ser “expresables” de modo “algorítmico”. Dicho de otro modo el conjunto de sucesiones de Cauchy es no numerable.

    Un saludo.

    ResponderEliminar
  27. José Luis, Enric:

    ´
    Antes de nada, gracias por esas dos respuestas excelentes, que además da la casualidad de que se complementan a la perfección. La de José Luis, con ese planteamiento tan visual, tan gráfico, de los intervalos que se van quedando escasos para alcanzar todos los irracionales, y la de Enric, con ese planteamiento algorítmico partiendo de las sucesiones de Cauchy. Las respuestas son claras, contundentes y no dejan lugar a dudas: hay números irracionales a los que no podemos ni siquiera aproximarnos con nuestros segmentillos.

    La perplejidad de mi último mensaje, sin embargo, tenía en su base una duda que sigue sin resolverse, y es la siguiente: ¿dónde están esos números inaproximables (llamémoslos así)? O, si se prefiere: ¿de qué manera se distribuyen? He pensado varias formas de hacer explícito a qué me refiero, y al final creo que la mejor es esta:
    Supongamos que retiramos de la recta real nuestros segmentillos y los irracionales que les sirven de límite. Es decir, supongamos que nos quedamos sólo con los números inaproximables. ¿Qué veríamos entonces? Si no me equivoco, la respuesta es que veríamos algo muy similar al conjunto de Cantor: una infinitud de puntos inconexos, una infinita sucesión de trozos de medida nula. Esto se debe, por supuesto, al hecho de que, dados dos números inaproximables cualesquiera, media entre ellos una separación de infinitos segmentillos.
    Por lo tanto, después de retirar nuestros segmentos del intervalo (0,1), el resultado sería un conjunto de medida nula.

    Un saludo

    ResponderEliminar
  28. Encuentro hoy una forma complementaria de plantear mi duda de ayer.

    Supongamos que, en esta ocasión, dejamos las sucesiones de segmentillos, junto con los irracionales que puedan hacer de límite, y lo que quitamos son los inaproximables. En este caso, lo que nos quedaría (si no me engaño) sería una hilera de segmentillos de medida no nula separados por huecos infinitesimales (puesto que, para el límite de cualquier sucesión, podemos encontrar otro tan arbitrariamente próximo como deseemos).

    Pues bien, la cuestión ahora es la siguiente: ¿cuántos puntos caben en esos huecos infinitesimales? En principio, yo diría que sólo uno. ¿Por qué? Porque, si fueran dos, entre ellos habría un racional con su correspondiente segmentillo, y por tanto ya no estarían los dos puntos en el mismo hueco, sino cada uno en uno distinto.

    Es decir, no hay forma de que en esos huecos haya más de un punto, de un número inaproximable, de donde se sigue que el conjunto de números sería tanto como el de huecos dejados por los intervalos: un conjunto numerable de medida cero. La única forma de evitar esto sería conseguir que al menos en alguno los huecos hubiera más de un punto (de hecho, debería haber aleph-1), pero esto no parece posible.

    ResponderEliminar
  29. Aunque intuyo que a estas alturas debo de ser el único que sigue por aquí, quiero intentar una demostración por reducción al absurdo de que la recta podría cubrirse por completo. La cosa, por pasos, sería así:

    1. Si los intervalos no han cubierto la recta, debe haber huecos.
    2. Un hueco puede definirse como un tramo de recta sin intervalos.
    3. En el hueco debe haber números irracionales (de hecho, debería haber un número incontable, pero esto no importa aquí).
    4. Entre dos números irracionales cualesquiera siempre hay algún racional.
    5. A cada racional le corresponde un intervalo.
    6. Por lo tanto, en cada hueco debe haber un intervalo, en contradicción con nuestra definición de "hueco" y con nuestro primer supuesto.

    ResponderEliminar
  30. Perplejo,

    Le voy a mostrar lo que yo creo es unl posible absurdo de su argumento, argumentando "ironicamente" del mismo modo que usted pero con los números racionales.

    Pero antes repito: Con los “ trocitos de recta (0,1) “ no se hace un cubrimiento de toda la recta, sino que simplemente se aprovecha la densidad de los racionales y el hecho de que (0,1) tiene el mismo cardinal que todo R, para establecer una correspondencia (que no cubrimiento) entre todo número real y los números reales del intervalo (0,1).

    Ahora la demostración:

    • Demostración de que los números racionales cubren toda la recta por reducción al absurdo:

    1. Si los racionales no cubren toda la recta debe haber huecos.

    2. Un hueco puede definirse como un tramo de recta sin racionales.

    3. Un hueco debe estar pues formado por números irracionales pero no racionales obviamente.
    4. Entre dos números irracionales cualesquiera siempre hay algún racional.
    5. Luego todo hueco tiene siempre un racional.
    6. Por lo tanto, todo hueco no es un hueco. (Absurdo !)
    7. luego los racionales cubren la recta.

    Un saludo.

    ResponderEliminar
  31. Enric Rodríguez:

    Déjeme empezar dándole las gracias y la enhorabuena por su ingeniosa respuesta: no es fácil concentrar más brillantez en menos líneas. Creo, sin embargo, que la cuestión queda abierta todavía, por razones tanto formales (digámoslo así) como de fondo.

    Las formales serían básicamente dos:

    1. Su reducción puede mostrar que hay un fallo en la mía, pero no indica dónde (yo, de un modo meramente intuitivo, sospecho de la identificación tácita que hago entre "tramo sin intervalos" y "tramo no cubierto por un intervalo", pero no he sido capaz de ir más allá).
    2. Para mostrar que algo no marcha en mi demostración no hace falta recurrir a una distinta: basta con la propia conclusión que se sigue del "teorema" de que un metro sería igual a infinito.

    Por lo que se refiere a las razones de fondo, la fundamental sería esta:

    En los términos en que usted plantea su demostración, cada número irracional es un hueco (puesto que no es un racional). Por lo tanto, cuando usted dice

    "4. Entre dos números irracionales cualesquiera siempre hay algún racional",

    lo que está diciendo es:

    "4. Entre dos huecos cualesquiera siempre hay algún racional",

    y de ahí no se sigue en modo alguno esto:

    "5. Luego todo hueco tiene siempre un racional".

    En el caso de mi "demostración", sin embargo, no detecto esta misma inconsistencia.

    Un saludo muy cordial

    ResponderEliminar
  32. Sr. Rodríguez:

    Olvide mi última objeción. No hay ninguna diferencia relevante entre esos pasos de su demostración y la mía. Los dos incidimos, de manera más o menos explícita, en lo mismo: el número de huecos parece numerable y cada hueco parece formado por un solo irracional, lo que parece llevar a un conjunto numerable de medida cero.

    Yo aventuraba ayer que el problema quizá se encuentre en mi definición de hueco, en la que identifico "tramo sin intervalos" con "tramo no cubierto por un intervalo". Otra posibilidad que se me ha pasado por la cabeza en otro momento era que el número de huecos no sea numerable, pero cada vez que lo pienso un poco me parece evidente que debe serlo.

    En fin, un misterio.

    Saludos

    ResponderEliminar
  33. Perplejo,

    “yo, de un modo meramente intuitivo, sospecho de la identificación tácita que hago entre "tramo sin intervalos" y "tramo no cubierto por un intervalo" “

    Exacto. Que una vez distribuidos los intervalos mediante la regla de traslación de los intervalos de (0,1) sobre los racionales ocurra que siempre cualquier intervalo de la recta que consideremos, por muy pequeño que sea, “Toque” (interseccione) con los intervalos de (0,1) trasladados, no significa que tal intervalo esté cubierto por estos. Tocar no significa recubrir. Y eso es igual de “extraño” que decir que en cualquier intervalo de la recta hay siempre números racionales, a pesar de tener estos medida cero y los irracionales medida infinita.

    la propia conclusión que se sigue del "teorema" de que un metro sería igual a infinito.

    ¿Dónde saca esa conclusión? Para poder decir que todos los intervalos trasladados miden infinito usted hace la siguiente suposición: Los trocitos Ei trasladaos miden 1 cuando están distribuidos de modo disjunto. Usted coloca los trocitos sobre los racionales, para aparentar que cubre la recta, (que no la cubre como le dije anteriormente, solo relaciona los puntos de la recta con los puntos del intervalo (0,1), es un asunto de cardinalidad de los reales y del intervalo (0,1) y de densidad de los racionales lo que se utiliza, no un cubrimiento topológico desde el cual definir la medida). Además, cuando usted traslada un intervalo de hecho dentro de ese intervalo hay infinitos intervalos más. Si traslada el intervalo (0, 1/2) al número racional q que le corresponda, ese nuevo intervalo tiene infinitos racionales q, los comprendidos entre (q-1/2, q+1/2). De manera que cada q de esos tiene también sus correspondientes trocitos de los segmentos de (0,1), de manera que no podemos definir qué medida tiene los intervalos distribuidos por los racionales a partir de la medida de los trocitos de (0,1) pues no son disjuntos, tenemos una nube de intervalos distribuidos unos dentro de otros de modo “caótico”.

    En los términos en que usted plantea su demostración, cada número irracional es un hueco (puesto que no es un racional).

    No. Usé la misma definición que usted, “Un hueco puede definirse como un TRAMO de recta sin racionales”.

    ”Los dos incidimos, de manera más o menos explícita, en lo mismo: el número de huecos parece numerable y cada hueco parece formado por un solo irracional, lo que parece llevar a un conjunto numerable de medida cero.”

    Nooo!!. ¿De dónde saca que yo digo eso? Lo que digo es que no hay huecos al modo que usted los define: “un hueco puede definirse como un tramo de recta que no intersecciones con ninguno de los intervalos trasladados” por el mismo modo que no hay un solo TRAMO de recta sin racionales.

    ”Otra posibilidad que se me ha pasado por la cabeza en otro momento era que el número de huecos no sea numerable, pero cada vez que lo pienso un poco me parece evidente que debe serlo.”

    Si entiende por hueco numero irracional está en lo cierto, los irracionales son innumerables (de hecho los transcendentes son los que ocasional la no numerabilidad) pero si entiende por hueco, TRAMO de recta sin racionales o tramo de recta que no intersecciones con alguno de los intervalos de (0,1) que estamos considerando distribuidos sobre los racionales, de eso ya le digo, no existe, pues los racionales son densos sobre R.

    Un saludo.

    ResponderEliminar
    Respuestas
    1. Sr. Rodríguez:


      Espero que me disculpe, pero no comprendo la distinción que hace entre "tocar" y "recubrir". Tal como yo lo entiendo, si trasladamos un segmento a la real, hemos recubierto en ella un segmento de idéntica longitud cuyo centro es un número racional. Y, si el teorema es cierto (es decir, si es un teorema), al final habremos recubierto la recta entera. O, si se prefiere, podríamos decirlo de esta forma algo enrevesada pero quizá más plástica: si el teorema es cierto, cualquier perpendicular que pase por un punto de la recta pasará también por algún punto de algún intervalo.

      Dice también que no podemos sacar la conclusión de que un metro es igual a infinitos, pero me parece que dice esto suponiendo que el teorema sea falso: yo digo que esto es una conclusión lógica si lo suponemos verdadero. Y añade esto:

      "Además, cuando usted traslada un intervalo de hecho dentro de ese intervalo hay infinitos intervalos más".

      Así es. En mensajes anteriores me he referido a ello hablando de "sucesiones de intervalos". Esto significa que no podemos saber la longitud exacta de nuestra recolocación de intervalos, pero sí podemos saber cuál debería ser su longitud máxima: un metro (que es lo que medía el segmento inicial). De ahí la sorpresa al comprobar que son capaces de cubrir infinitos metros (o cualquier otra longitud).

      "Nooo!!. ¿De dónde saca que yo digo eso?"

      No, no digo que lo diga usted. Ya sé que su postura es la contraria: digo que es lo dice su demostración irónica, su reducción al absurdo de mi reducción al absurdo.

      Un saludo

      Eliminar
  34. Perplejo,

    no comprendo la distinción que hace entre "tocar" y "recubrir".

    El segmento (0,3) recubre al segmento (1,2), pero el segmento (1,2) o el (2,4) solo “tocan al segmento (0,3) . Tocar significa simplemente que interseccionan, que tienen elementos en común, pero no todos.

    Tal como yo lo entiendo, si trasladamos un segmento a la real, hemos recubierto en ella un segmento de idéntica longitud cuyo centro es un número racional.

    Exacto

    Y, si el teorema es cierto (es decir, si es un teorema), al final habremos recubierto la recta entera.

    No, el teorema sólo puede decir que no hay ningún segmento de la recta real por pequeño que sea que no quede “Tocado” por los segmentos de (0,1) trasladados, pero como dije más arriba, “tocado” no es lo mismo que recubierto.


    si el teorema es cierto, cualquier perpendicular que pase por un punto de la recta pasará también por algún punto de algún intervalo.

    No. Cualquier perpendicular de esas puede pasar por infinidad de puntos que no son de los intervalos. Lo que está diciendo el teorema, tal como yo lo interpreto, es que hemos distribuido los infinitos intervalos Ei de (0,1) , formando los intervalos Qi ( los Ei trasladados) DE TAL MANERA QUE NO HAY NINGÚN INTERVALO DE LA RECTA (POR MUY PEQUEÑO QUE SEA) QUE PUEDA DECIR QUE EN SU SENO NO HAY ELEMENTOS DE LOS Qi. Pero que haya elementos de Qi en todo intervalo de la recta no significa que la recta este recubierta por los Qi.

    Dice también que no podemos sacar la conclusión de que un metro es igual a infinitos, pero me parece que dice esto suponiendo que el teorema sea falso: yo digo que esto es una conclusión lógica si lo suponemos verdadero.

    Tal como interpreto yo el teorema, no. Si usted cree que el teorema dice que con los Ei cubrimos la recta real, pues entonces lo que digo es que el teorema es falso. Pero repito, me parece que el teorema no dice eso.

    sigo...

    ResponderEliminar
  35. ...sigo

    “Esto significa que no podemos saber la longitud exacta de nuestra recolocación de intervalos

    Eso digo.

    De ahí la sorpresa al comprobar que son capaces de cubrir infinitos metros (o cualquier otra longitud).

    Insisto una vez más, NO SE RECUBRE LA RECTA. Solo se dice que no hay un solo trozo de recta que quede “salvado” de no ser tocado por los infinitos y numerables segmentos de (0,1) trasladaos sobre los racionales, simplemente porque los racionales son densos en R.


    “No, no digo que lo diga usted. Ya sé que su postura es la contraria: digo que es lo dice su demostración irónica, su reducción al absurdo de mi reducción al absurdo.”


    No entiendo, mi demostración irónica no dice que : “el número de huecos parece numerable y cada hueco parece formado por un solo irracional, lo que parece llevar a un conjunto numerable de medida cero” .

    Mi demostración solo dice una perogrullada; a saber, que si “hueco” es un “TRAMO de recta sin racionales” es lógico que no haya huecos, pues los racionales son densos y no puede haber trozos de recta solo formados por irracionales. Y que del mismo modo, si usted define un hueco (así lo hizo) como un “tramo de la recta que no sea “tocado” por alguno de los infinitos segmentos trasladados de (0,1)” entonces tampoco hay huecos tal como usted los define. La ironia intenta mostrar que la causa de ello es la densidad de Q sobre R . Que en el fondo lo que se hace al trasladar los segmentos es eso, establecer que no hay trozo de la recta, por pequeño que sea, que no tenga números racionales ( y por tanto un segmento de (0,1) asociado). ¡Pero eso no significa que cubramos toda la recta !, como mucho, podremos cubrir un conjunto de medida 1 (la longitud del intervalo). Y esto es parecido a lo que ocurre si ententamos cubrir la recta sólo con los números racionales. Los racionales tampoco la cubren, obviamente. Los racionales tienen medida cero, y los segmentos de (0,1) medida 1. Los racionales son densos en R, y “los segmentos de (0,1) TRASLADADOS sobre los racionales“ también (entre dos reales siempre hay un segmento de esos) pero eso no significa, repito, ni que Q ni que los segmentos esos cubran toda la recta.

    Un saludo

    ResponderEliminar
  36. Sr. Rodríguez:


    Creo que ya comprendo su distinción entre "tocar" y "cubrir". Una cosa, en efecto, es que no haya un tramo que no contenga intervalos y otra muy distinta, que el tramo esté cubierto por completo. La distinción, sin embargo, no cambia lo esencial: que los huecos de la recta no son tramos, sino puntos. Es decir, no cambia el hecho de que nunca encontraremos una sucesión de puntos que podamos llamar "hueco". Los huecos, a lo sumo, tienen un solo punto y son de medida nula.

    Por decirlo gráficamente, una vez realizada la traslación del teorema o pseudoteorema, nos quedaría una especie de morse infinito donde los tramos cubiertos completamente por intervalos son las rayas y los huecos serían los puntos, formados cada uno de ellos por un único irracional. Y, dado que el número de huecos es numerable (puesto que sólo puede haber dos, uno por cada extremo de los tramos o rayas) su medida sería nula. De hecho, sería nula incluso si el número de huecos fuera no numerable, pues nos encontraríamos ante un conjunto de puntos equivalente al polvo de Cantor.

    Un saludo







    ResponderEliminar
    Respuestas
    1. Habría que hacer una observación acerca de la naturaleza de los puntos que no quedan cubiertos por los intervalos. Quizá luego pueda añadir algo.

      Eliminar
  37. En un mensaje anterior distinguíamos dos tipos de irracionales: los que podían ser aproximados mediante sucesiones de intervalos y los que no. Los primeros estaba claro dónde podíamos encontrarlos (en los extremos de cada sucesión de intervalos, precisamente), pero los segundos eran un misterio, al menos para mí.

    Pensando un poco en el asunto, creo que intuyo dónde pueden estar, y la respuesta da idea de la asombrosa complejidad que oculta la aparente monotonía de la recta real.

    En pocas palabras, la clave estaría en un jerarquía infinita de sucesiones. En un primer nivel estarian los intervalos, cuyos extremos son números racionales. Los intervalos se agrupan en sucesiones (o nubes caóticas, más bien, como las describió el señor Rodríguez) cuyos extremos son números irracionales "aproximables". Aquí me había quedado yo. Parece esperable (aunque yo no sería capaz de demostrarlo en este momento) que esas sucesiones o nubes de intervalos se agrupen en supersucesiones cuyo límite serían los inaproximables. Y estas supersucesiones, por supuesto...

    Yo imagino que la clave de la paradoja en esta estructura. Aparentemente estaría ese morse de puntos y rayas del que mencionaba en mi respuesta al señor Rodríguez, pero, visto en detalle, no todas sus rayas y puntos son iguales. Aparte, esto permitiría la presencia de un conjunto no numerable de puntos, y no meramente numerable.

    ResponderEliminar
  38. Hola, Perplejo. Sí, algo así pienso yo, que el mismo conjunto numerable de intervalos con el que hemos cubierto cada número racional de la recta puede descomponerse en una cantidad innumerable de series de intervalos, cada una de los cuales tendría como límite un número irracional. Pero aún así me gustaría ver una prueba más detallada de cómo sería esto posible.
    Un saludo

    ResponderEliminar

Deja aquí tu comentario (comentarios moderados)